A skating rink charges a group rate of $9 plus a fee to rent each pair of skates. A family rents 7 pairs of skates and pays a total of $30. Draw a tape diagram

Answers

Answer 1

Answer:

X = 3

Step-by-step explanation:

I can't really draw the diagram for you.

$9 is always charged so just add that to the end of your equation.

x is what they charge for skates and their are 7 skates so 7x

$30 is the total

7x + 9 = 30

subtract 9 from both sides

7x = 21

divide by 7 on both sides

x = 3


Related Questions

Solution of inequality ((x - 1)(x - 5))/(x - 3) > 0

Answers

Answer:

  (1, 3) ∪ (5, ∞)

Step-by-step explanation:

You want the solution to the inequality ((x -1)(x -5))/(x -3) > 0.

Sign changes

The sign of the function changes at values of x that make the factors zero, at x = 1, 3, 5. The function is positive for x > 5, so will also be positive for 1 < x < 3

The solution is ...

  1 < x < 3  or  5 < x

__

Additional comment

If any linear factor has an even degree (even multiplicity), there will not be a sign change. The numerator factors correspond to function zeros. The denominator factors correspond to function vertical asymptotes.

The attached graph shows zeros at x=1 and x=5, and a vertical asymptote with a sign change at x=3.

What is 252 divided by 9

Answers

Answer:

28 is the ans of 252÷9here we go

252 divided by 9 equals 28.

To divide 252 by 9, you can use long division, which involves dividing the number in steps until there is no remainder left.

Here's the step-by-step process:

Write down the dividend (252) and the divisor (9), and set up the long division format:

9 | 252

Look at the leftmost digit of the dividend (2) and see if it's divisible by the divisor (9). Since 2 is less than 9, we bring down the next digit (5) to the right of 2, making it 25.

9 | 252

2

Divide the new number (25) by the divisor (9). The result is 2, which is the first digit of the quotient. Multiply this result by the divisor (2 x 9 = 18) and write it below the 25, then subtract it from 25:

9 | 252

25

18

--

7

Bring down the next digit (2) from the dividend to the right of the remainder (7), making it 72. Now, divide 72 by 9, which gives you 8. Multiply this result by the divisor (8 x 9 = 72) and write it below the 72, then subtract it from 72:

9 | 252

25

18

--

72

72

---

0

There is no remainder left, and the dividend has been completely divided. The quotient is the result of the division, which is 28.

Therefore, 252 divided by 9 equals 28.

Learn more about long division click;

https://brainly.com/question/28824872

#SPJ6

n one region of the country, the mean length of stay in hospitals is 5.5 days with standard deviation 2.6 days. Because many patients stay in the hospital for considerably more days, the distribution of length of stay is strongly skewed to the right. Consider random samples of size 100 taken from the distribution with the mean length of stay, x, recorded for each sample. Which of the following is the best description of the sampling distribution of x ?a.Strongly skewed to the right with mean 5.5 days and standard deviation 2.6 daysb, Strongly skewed to the right with mean 5.5 days and standard deviation 0.26 dayc. Strongly skewed to the right with mean 5.5 days and standard deviation 0.026 dayd. Approximately normal with mean 5.5 days and standard deviation 2.6 dayse. Approximately normal with mean 5.5 days and standard deviation 0.26 day

Answers

(e): Approximately normal with mean 5.5 days and standard deviation 0.26 days.

What is standard deviation?

Standard deviation is a measure of the spread or dispersion of a set of data from its mean or average. It measures how far the values in a dataset are from the mean of the dataset.

The sample size needed for this approximation to hold depends on the shape of the population distribution, but a commonly used rule of thumb is that a sample size of at least 30 is sufficient for most cases.

In this case, the sample size is 100, which is larger than 30, so we can use the central limit theorem to approximate the sampling distribution of the sample mean.

The mean of the population distribution is 5.5 days, and the standard deviation is 2.6 days. The standard deviation of the sampling distribution of the sample mean can be approximated using the formula:

standard deviation of the sample mean = standard deviation of the population / square root of sample size

= 2.6 / sqrt(100)

= 0.26

Therefore, the best description of the sampling distribution of x is option (e): approximately normal with mean 5.5 days and standard deviation 0.26 days.

To learn more about standard deviation visit:

https://brainly.com/question/475676

#SPJ1

John Paszel took out a loan for 51 months but paid it in full after 25 months. Find the refund fraction he should use to
calculate the amount of his refund.

Answers

Answer: calculate the amount of his refund.

Step-by-step explanation:

The blueprint of a triangular patio has side lengths 3.5 in., 3.5 in., and 5.25 in. If the shorter sides of the actual patio are each 10.5 ft long, how long is the third side?

Answers

The length of the third side of the actual patio is 15.75 feet.

What is similarity of triangle?

In order to solve issues requiring proportional connections between the sides of two triangles, it is crucial to understand the idea of triangle similarity. If the matching sides and angles of two triangles are identical, then two triangles are said to be similar. When referring to similarities, the symbol is used.

Let us suppose the length of third side = x.

Given that, blueprint of a triangular patio has side lengths 3.5 in., 3.5 in., and 5.25 in.

Thus, the blueprint and the actual patio form two similar triangles.

The sides of the similar triangles are equal in ratio thus,

3.5/5.25 = 10.5/x

3.5x = 5.25 * 10.5

x = (5.25 * 10.5) / 3.5

x = 15.75

Hence, the length of the third side of the actual patio is 15.75 feet.

Learn more about congruency here:

https://brainly.com/question/7888063

#SPJ1

Given ΔABC with measure of angle B equals 78 degrees, measure of angle C equals 52 degrees, and a = 16 inches, what is the length of b?

Answers

To find the length of side b in triangle ABC, we can use the Law of Sines. The length of side b is approximately 20.058 inches.

To find the length of side b in triangle ABC, we can use the Law of Sines. The Law of Sines states that the ratio of the length of a side to the sine of its opposite angle is constant for all sides and angles in a triangle.

Using the Law of Sines, we have:

sin(A)/a = sin(B)/b

We are given the measure of angle B as 78 degrees and side a as 16 inches. We can substitute these values into the equation:

sin(A)/16 = sin(78)/b

To find sin(A), we can use the fact that the sum of the angles in a triangle is 180 degrees:

A + B + C = 180

A + 78 + 52 = 180

A = 180 - 78 - 52

A = 50 degrees

Now we can substitute the values into the equation again:

sin(50)/16 = sin(78)/b

To solve for b, we can cross-multiply and isolate b:

b = (16 * sin(78))/sin(50)

We can calculate the length of side b by evaluating this expression using a calculator. The measurement will be roughly 20.058 inches.

for such more question on length

https://brainly.com/question/24176380

#SPJ8

Write the HCF of x
3y
4z
2 and x
2y
3z
5, where x, y, z are
distinct prime numbers

Answers

the HCF of x, 2y, 3y, 4z, x², 3z, and 5, where x, y, z are

distinct prime numbers is 1.

To find the highest common factor (HCF) of the given numbers, we need to find the common factors of each pair of numbers and then find the highest common factor of all the resulting common factors.

First, let's find the prime factors of the given numbers:

x = a prime number (distinct from y and z)

2y = 2 × y

3y = 3 × y

4z = 2² × z

3z = 3 × z

x² = a prime number squared (distinct from y and z)

5 = a prime number

Next, we can pair up the numbers and find their common factors:

Common factors of x and 2y: 1, 2, y

Common factors of 3y and 4z: 1, 2, 3, y, z, 6

Common factors of x² and 3z: 1, 3, x, z, xz

Common factors of 5 and 2: 1

Finally, we find the highest common factor of all the resulting common factors:

The highest common factor of x, 2y, 3y, 4z, x², 3z, and 5 is 1, since it is the only factor that is common to all the pairs.

Therefore, the HCF of x, 2y, 3y, 4z, x², 3z, and 5 is 1.

To learn more about prime numbers click here

brainly.com/question/30358834

#SPJ4

a new surgery is successful of the time. if the results of such surgeries are randomly sampled, what is the probability that at least of them are successful? carry your intermediate computations to at least four decimal places, and round your answer to two decimal places. (if necessary, consult a list of formulas.)

Answers

The probability that at least one surgery is successful, given that a new surgery is successful 80% of the time, is 99.20%.

Let X denote the number of successful surgeries out of n randomly selected surgeries. The random variable X follows a binomial distribution. The probability of X = x successful surgeries out of n randomly selected surgeries is given by:

P(X=x)=nCxp^x(1−p)^n−x

where p is the probability that a new surgery is successful, and (1-p) is the probability that a new surgery is unsuccessful. In this case, p = 0.80 and (1-p) = 0.20. Let n = 5. Then, we have:

P(X≥1)=1−P(X=0)=1−nC0p0(1−p)n−0

=1−(0.20)5

=0.9936 ≈ 99.20%

Therefore, the probability that at least one surgery is successful is 99.20%.

To learn more about probability refer :

https://brainly.com/question/29350029

#SPJ11

Armando opened a new savings account with an initial deposit if $250. Which combination will result in a zero balance in armandos account?

Answers

A zero balance in Armando's account can be achieved by subtracting $250 from the account balance. This can be done through a combination of withdrawals and/or transfers out of the account.

What is subtracting?

Subtracting is a mathematical operation that involves taking one number away from another. It is the inverse of addition, and is represented by the minus sign (-). Subtracting is used to determine the difference between two numbers, or to reduce a number by a given amount. In algebraic terms, subtracting is represented by the equation: a - b = c, where a is the starting number, b is the number to be subtracted, and c is the resulting difference. Subtracting can also be used on fractions, decimals, and other forms of numbers.

For example, Armando could make a $250 withdrawal from the account or transfer $250 to another bank account.

To learn more about subtracting

https://brainly.com/question/17301989

#SPJ1

how many 5-letter passords can be made using the letters a thought z if... a) repetition of letters is allowed? b) repetition of letters is not allowed?

Answers

Answer:

Order is important: If allowed 11881376

Order is important:If rep. Is not allowed 7893600

Step-by-step explanation:

If the order isnt important and repetition is not allowed 65780

If order isn’t important and repetition is allowed 142506

4. What is the solution to 2 + 3(2a + 1) = 3(a + 2)?

Answers

Answer:

a=1/3

Step-by-step explanation:

First, expand the brackets by doing multiplication:

2+6a+3=3a+6

Then, move the unknown to the left and the numbers to the right:

3a=6-5

3a=1

a=1/3

The solution to the given equation is -1.

What is an equation?

In mathematics, an equation is a formula that expresses the equality of two expressions, by connecting them with the equals sign =.

The solution of an equation is the set of all values that, when substituted for unknowns, make an equation true.

The given equation is 2+3(2a+1)=3(a+2)

2+6a+3=3a+2

6a+5=3a+2

6a-3a=2-5

3a=-3

a=-1

Therefore, the solution to the given equation is -1.

To learn more about an equation visit:

https://brainly.com/question/14686792.

#SPJ2

plese help with the following questions PLESE
Assignment:
Sebastian Salazar opened a Sporting Goods Store. He has some new inventory that needs to be priced. He also has some seasonal inventory that hasn’t sold and he needs to put them on sale.
Help Sebastian with his pricing questions:

Answers

1. The retail prices are: $44.36, $34.10 and $67.57

2. The sales prices are: $263.20 and $65.59

Define the term retail price?

Retail price refers to the price at which a product or service is sold to the final consumer by a retailer. This price includes the cost of production, transportation, marketing, and other related expenses, as well as a markup to generate profit for the retailer.

1. New Items - Calculate Retail Prices

  Item                    Wholesale        Mark Up       Calculation              Retail

                              Cost ($)            Percent%                                      Price ($)

a. Louisville             $22.75             95%          22.75+(95%×22.75)  =$44.36

Slugger bat and

batting glove set

b. Wilson                 $11.00                210%         11+(210%×11)              =$34.10

Basketball

c. Element               $26.50              155%      26.50+(155%×26.50) =$67.57

Skateboard

2. Items on sale - Calculate Sale Prices

    Item                 Retail             Discount           Calculation            Sales

                           price ($)          Percent %                                       Price ($)

a. Head              $329.00              20%          329-(20%×329)         $263.20

Snowboard

b. Kelly                $79.99                18%           79.99-(18%×79.99)     $65.59

Ski poles

All data is filled in below table.

To know more about sales price, visit:

https://brainly.com/question/26008313

#SPJ1

Jayden evaluated the expression a + (2 + 1. 5) for a = 14. He said that the value of the expression was 8. 5. Select all the statements that are true. Jayden's solution is incorrect. Jayden added inside the parentheses before dividing. Jayden substituted the wrong value for a. Jayden divided 14 by 2 and then added 1. 5. Jayden added inside the parentheses before multiplying. ​

Answers

It is true that Jayden's solution is incorrect. It is false that Jayden added inside the parentheses before dividing.

It is false that Jayden substituted the wrong value for a. It is true that Jayden divided 14 by 2 and then added 1. 5. Jayden added inside the parentheses before multiplying. ​

1) The correct solution is

Given,

a ÷ (2 + 1. 5)

Substituting the value of a which is 14

= 14 ÷ (2 + 1. 5)

= 14 ÷ 3.5

= 4

2) As there is no term which needs to be divided so, the second statement is false.

3) Jayden didn't substitute the wrong value of a he just solved the given expression without considering the bracket and divided the 14 which is the value of a by 2.

4) Jyaden divided 14 by 2 and then added 1. 5. Jayden added inside the parentheses before multiplying. ​

i.e. a ÷ (2 + 1. 5)

14 ÷ 2 + 1. 5

7+1.5

8.5

This is the way Jayden solved the equation due to which he arrived at the wrong solution.

Learn more about Expression here: brainly.com/question/14083225

#SPJ4

The Correct question is as below

Jayden evaluated the expression a ÷ (2 + 1.5) for a = 14. He said that the answer was 8.5. Choose True or False for each statement.

1. Jayden's solution is incorrect.

2. Jayden added in the parentheses before dividing.

3. Jayden substituted the wrong value for a.

4. Jayden divided 14 by 2 and added 1.5

On a particularly strange railway line, there is just one infinitely long track, so overtaking
is impossible. Any time a train catches up to the one in front of it, they link up to form a
single train moving at the speed of the slower train. At first, there are three equally spaced
trains, each moving at a different speed. After all the linking that will happen has happened,
how many trains are there? What would have happened if the three equally spaced trains
had started in a different order, but each train kept its same starting speed? On average
(where we are averaging over all possible orderings of the three trains), how many trains will
there be after a long time has elapsed? What if at the start there are 4 trains (all moving
at different speeds)? Or 5? Or n? (Assume the Earth is flat and extends

Answers

After all the linking that will happen, there will be only one train left, moving at the speed of the slowest initial train.

If the three equally spaced trains had started in a different order, the final configuration would still be the same - one train moving at the speed of the slowest initial train.

On average, there will always be only one train left, regardless of the initial ordering of the trains. This is because after all the linking, the train with the slowest speed will always be at the front of the train and no other train can pass it.

If there are four equally spaced trains initially, after all the linking, there will be one train left moving at the speed of the slowest initial train. This is because any two faster trains that link up will then be slower than the train behind them, so the only train left will be the slowest one.

Similarly, if there are five or more equally spaced trains initially, after all the linking, there will be only one train left moving at the speed of the slowest initial train.

Therefore, regardless of the number of equally spaced trains initially, there will always be only one train left after all the linking.

To learn more about “average” refer to the :https://brainly.com/question/130657

#SPJ11

Consider the following differential equations. Determine if the Existence and Uniqueness Theorem does or does not guarantee existence and uniqueness of a solution of each of the following initial value problems.{eq}\begin{array}{l}{\frac{d y}{d x}=\sqrt{x-y}, \quad y(2)=2} \\ {\frac{d y}{d x}=\sqrt{x-y}, \quad y(2)=1} \\ {y \frac{d y}{d x}=x-1, \quad y(0)=1} \\ {y \frac{d y}{d x}=x-1, \quad y(1)=0}\end{array} {/eq}

Answers

Existence and Uniqueness Theorem the existence and uniqueness theorem is the most critical theorem in differential calculus. The theorem addresses how the existence and uniqueness of a solution to a first-order differential equation are affected by conditions such as continuity or Lipschitz continuity.

Determine if the existence and uniqueness theorem does or does not guarantee the existence and uniqueness of a solution to each of the following initial value problems.

1. The differential equation is

                     [tex]\frac{dy}{dx}=\sqrt{x-y}[/tex]

       The condition of the theorem is fulfilled: The differential equation is continuous and the partial derivative   [tex]\frac{\partial f}{\partial y}=\frac{-1}{2\sqrt{x-y}}[/tex]  is continuous.

            Therefore, the theorem guarantees the existence and uniqueness of the solution to the initial value problem.

2. The differential equation is

              [tex]\frac{dy}{dx}=\sqrt{x-y}[/tex]

         The condition of the theorem is not fulfilled.

            [tex]\frac{\partial f}{\partial y}=\frac{-1}{2\sqrt{x-y}}[/tex] is not defined at [tex]x=y .[/tex]

            Therefore, the theorem does not guarantee the existence and uniqueness of the solution to the initial value problem.

3. The differential equation is  [tex]y\frac{dy}{dx}=x-1[/tex] The condition of the theorem is fulfilled: The differential equation is continuous and the partial derivative [tex]\frac{\partial f}{\partial y}=y[/tex] is continuous.

     Therefore, the theorem guarantees the existence and uniqueness of the solution to the initial value problem.

4. The differential equation is

              [tex]y\frac{dy}{dx}=x-1[/tex]

       The condition of the theorem is not fulfilled   [tex]\frac{\partial f}{\partial y}=y[/tex]  and is not defined at  y=0

    Therefore, the theorem does not guarantee the existence and uniqueness of the solution to the initial value problem.

Learn more about the Existence and Uniqueness Theorem at: https://brainly.com/question/30417555

#SPJ11

Give an algebraic proof for the triangle inequality ||v + w| < ||v|| + ||w|| Draw a sketch. Hint: Expand ||v + w||^2 = (v+w) · (v + w). Then use the Cauchy-Schwarz inequality.

Answers

An algebraic proof of the triangle inequality is ||u + v|| ≤ ||u|| + ||v||.

Let u and v be vectors in a vector space.

Using the properties of the inner product,

we have[tex],|u + v|² = (u + v) · (u + v)= u · u + u · v + v · u + v · v= |u|² + 2u · v + |v|²[/tex]

Now, take the square root of both sides of the above equation to get

|u + v| = sqrt(|u|² + 2u · v + |v|²) ≤ sqrt(|u|² + 2|u||v| + |v|²)= sqrt(|u|²) + sqrt(2|u||v|) + sqrt(|v|²) = |u| + |v|

Therefore, ||u + v|| ≤ ||u|| + ||v||. The equality holds if and only if u and v are linearly dependent.

Hence, the inequality is sometimes called the strict triangle inequality.

for such more questions on  algebraic

https://brainly.com/question/395066

#SPJ11

Find the volume of the cylinder given the height and radius shown.
14
21

Answers

Answer:

volume=12930.79

Step-by-step explanation:

[tex]v=\pi r^{2} h\\v=\pi (14)^{2} (21)\\v=12930.79[/tex]

WILL YOU CRACK THE CODE ? 8 2 One number is correct and well placed One number is correct but wrong place Two numbers are correct but wrong places 3 8 Nothing is correct CODE 8 One number is correct but wrong place

Answers

Cracking the code: 8 2One number is correct and well placed One number is correct but in the wrong placeTwo numbers are correct but in the wrong place38Nothing is correctCODE8One number is correct but in the wrong placeCracking the code of this sequence of numbers can be a bit tricky, but let's do it step by step. We are given the following clues about the sequence of numbers:

One number is correct and well placed: Since the sequence of numbers is 8 2, we know that the number 8 is in the first position. So the code is either 8 _ _ _ or _ _ _ 8.One number is correct but in the wrong place: This clue tells us that the number 2 is not in the second position of the code, but it is somewhere else.

Therefore, we know that the code is not 8 2 _ _ or _ _ 2 8.Two numbers are correct but in the wrong places: This clue tells us that the code contains the numbers 3 and 8, but they are in the wrong position. Since the code cannot be 8 2 _ _ or _ _ 2 8, we know that the two correct numbers are not in the last two positions. Therefore, the code must be _ 3 8 _ or _ 8 3 _.Nothing is correct: This clue tells us that the code cannot be 3 8 _, 8 3 _, or _ 3 8 because they all contain at least one correct number. Therefore, the code must be _ _ 3 8 or 3 8 _ _.One number is correct but in the wrong place: This clue tells us that the code cannot be 3 8 _, so it must be 8 3 _. Therefore, the code is 8 3 _ _.I hope this helps you crack the code!

for such more questions on sequence of numbers

https://brainly.com/question/30983233

#SPJ11

pls help urgent




which expression can be used to find the surface area??(see pic)

Answers

An expression which can be used to find the surface area of the triangular prism include the following: C. 2(1/2 · 3 · 4) + 4 · 2 + 3 · 2 + 5 · 2.

How to calculate the surface area of the triangular prism?

In Mathematics, the surface area of a triangular prism can be calculated by using this mathematical expression:

Total surface area of triangular prism = (Perimeter of the base × Length of the prism) + (2 × Base area)

Total surface area of triangular prism = (S₁ + S₂ + S₃)L + bh

where:

b represent the bottom edge of the base triangle.h is the height of the base triangle.L represent the length of the triangular prism.S₁, S₂, and S₃ represent the three sides (edges) of the base triangle.

By substituting the given parameters into the surface area of triangular prism formula, we have:

Surface area of triangular prism = Area (GFEH + BCEF + BIJC + ABF + ECD)      

Surface area of triangular prism = 4 · 2 + 3 · 2 + 5 · 2 + (¹/₂ · 3 · 4) + (¹/₂ · 3 · 4)  

Surface area of triangular prism = 2(1/2 · 3 · 4) + 4 · 2 + 3 · 2 + 5 · 2.

Read more on triangular prism here: brainly.com/question/21173224

#SPJ1

twenty-five percent of the employees of a large company are minorities. a random sample of 7 employees is selected. what is the probability that the sample contains exactly 4 minorities?

Answers

The probability that the sample contains exactly 4 minorities is 0.0577

How to find the probability?

We know that 25% of the employees of a large company are minorities. So, from any set of N employees that we ranodmly select from that large company, we can estimate that the number of minorities there is:

0.25*N

Now, we want to find the probability that the sample contains exactly 4 minorities.

Then we need to compute:

[tex]P = 0.25^4*0.75^3[/tex]

We also need to count the permutations for that probability, then we need to add the factor:

[tex]P = 0.25^4*0.75^3*(\frac{7!}{4!*3!} )[/tex]

Solving that we will get:

[tex]P = 0.25^4*0.75^3*(\frac{7!}{4!*3!} )\\\\P = 0.25^4*0.75^3*(\frac{7*6*5}{3*2} )\\\\P = 0.0577[/tex]

That is the probability.

Learn more about probability at:

https://brainly.com/question/25870256

#SPJ1

The angles opposite the congruent sides of an isosceles triangle are congruent. Find the value of x in the triangle. Show all your work.


The figure shows an angle with a measure of 52 degrees.
A. Find the complement of the angle shown.
B. Find the supplement of the angle shown.
Show all your work.

Answers

The complement of the angle is 38 degrees and the supplement of the angle is 128 degrees.

Find the complement of the angle shown.

The given angle is

Angle = 52 degrees

To find the complement of an angle, we need to subtract its measure from 90 degrees.

Therefore, the complement of an angle with a measure of 52 degrees is:

90 degrees - 52 degrees = 38 degrees

So, the complement of the angle shown is 38 degrees.

Find the supplement of the angle shown.

To find the supplement of an angle, we need to subtract its measure from 180 degrees.

Therefore, the supplement of an angle with a measure of 52 degrees is:

180 degrees - 52 degrees = 128 degrees

So, the supplement of the angle shown is 128 degrees.

Read more about angles at

https://brainly.com/question/25716982

#SPJ1

Use the equation to explain why the rule is a linear function

Answers

The linear function rule is   y = (1/5)x - 5 .

What is a linear equation, exactly?

An algebraic equation with simply a constant and a first-order (linear) term, such as y=mx+b, where m is the slope and b is the y-intercept, is known as a linear equation.

                             Sometimes, the aforementioned is referred to as a "linear equation of two variables," where x and y are the variables. There are numerous instances of linear equations, including 2x - 3 = 0, 2y = 8, m + 1 = 0, x/2 = 3, x + y = 2, and 3x - y + z = 3.

The graph of a linear function is a straight line.  

f(0) = -5 means that (0,-5) is a point on the line.  Similarly, (-5,-6) is also a point on the line.  

Slope of line = m = [-6-(-5)]/[-5-0] = -1/(-5) = 1/5

Using the point-slope form, an equation of the line is:

y - (-5) = (1/5)(x - 0)

So, y = (1/5)x - 5

Learn more about linear function

brainly.com/question/20286983

#SPJ1

The complete question is -

Write a rule for a linear function y=f (x), given that f (0) =-5 and f (-5) =-6. Write answers using fractions or integers.

The function rule is f(x) =

if you randomly select a card from a well-shuffled standard deck of 52 cards, what is the probability that the card you select is a diamond or 9? (your answer must be in the form of a reduced fraction.)

Answers

The probability of selecting a diamond or 9 is 1/13.

Step by step explanation: There are 4 suits (diamonds, clubs, hearts, and spades) and 13 ranks (2, 3, 4, 5, 6, 7, 8, 9, 10, Jack, Queen, King, and Ace). The probability of selecting a card from a standard deck of 52 cards is 1/52.

To calculate the probability of selecting a diamond or a 9, you must first calculate the total number of diamonds and nines in the deck, which is 8 (4 diamonds and 4 nines). The probability of selecting a diamond or 9 is 8/52, which reduces to 1/13.

See more about probability at: https://brainly.com/question/24756209

#SPJ11

Sarah is a healthy baby who was exclusively breast-fed for her first 12 months. Which of the following is most likely a description of her weights (at 3, 6, 9, and 12 months of age) as percentiles of the CDC growth chart reference population? 85th percentile at 3 months; 85th percentile at 6 months; 9oth percentile at 9 months; 95th percentile at 12 months 75th percentile at 3 months; 40th percentile at 6 months; 25th percentile at 9 months; 25th percentile at 12 months 30th percentile at 3 months; 50th percentile at 6 months; 70th percentile at 9 months; 80th percentile at 12 months 25th percentile at 3 months; 25th percentile at 6 months; 25th percentile at 9 months; 25th percentile at 12 months

Answers

The 12 months of age) as percentiles of the CDC growth chart reference population.

The most likely description of Sarah's weights (at 3, 6, 9, and 12 months of age) as percentiles of the CDC growth chart reference population is: 85th percentile at 3 months; 85th percentile at 6 months; 90th percentile at 9 months; 95th percentile at 12 months.What is percentile in statistics?In statistics, a percentile is a value below which a specific percentage of observations in a group falls. It is used to split up data into segments that represent an equal proportion of the entire group, resulting in a data set split into 100 equal portions, with each portion representing one percentage point. Sarah's weight is in the 85th percentile at 3 months, 85th percentile at 6 months, 90th percentile at 9 months, and 95th percentile at 12 months is a most likely description of her weights (at 3, 6, 9, and 12 months of age) as percentiles of the CDC growth chart reference population.

Learn more about Percentiles

brainly.com/question/28800128

#SPJ11

What is the percent change of 31 to 7

Answers

The percent change of 31 to 7 is found as  77.42% which is percent decrease of the number.

Explain about percent change?

You can determine the amount of change that has occurred over this time period if you have data at two points in time. The outcome is known as the rate of change, sometimes known as the percentage change, and is given in percentage form (in absolute numbers, it really is merely a difference).

Using a percentage is one method of calculating a percent of change. Keep in mind that a percentage indicates "per 100." You can rephrase your problem as a % out of 100 by first describing the change as a fraction and measuring the amount of change to the original.

Original number = 31

Final number = 7

percent change = (7 - 31)/31 *100

percent change = - 24*100 / 31

percent change = - 77.42

Here negative sign shows the percent decrease of the number.

Know more about the percent change

https://brainly.com/question/105388

#SPJ1

please help fast I am baffled

Answers

The expression for the number of non-adult sizes is s - 19.

What are expressions?

A value or amount is represented by an expression, which is a collection of numbers, variables, and mathematical operations like addition, subtraction, multiplication, and division. Calculations, complicated mathematical equations, and issues in a variety of disciplines, including science, engineering, economics, and statistics, may all be solved using expressions. Functions that depict a connection between variables, such as sin(x) and log(x), can also be included in expressions. Expressions are frequently employed to simulate real-world circumstances and provide predictions based on mathematical analysis.

Given that the total number od sweatshirts = s.

The number of non-adult sweatshirts can be calculated by:

Number of non-adult sizes = Total number of sweatshirts sold - Number of adult sizes

= s - 19

Hence, the expression for the number of non-adult sizes is s - 19.

Learn more about mathematical expressions here:

https://brainly.com/question/28980347

#SPJ1

IN A BOX PLOT , IF THE MEDIAN IS TO THE LEFT OF THE CENTER OF THE BOX AND THE RIGHT WHISKER IS SUBSTANTIALLY LONGER THAN THE LET WHISKER, THE DISTRIBUTION IS SKEWED LEFT OR RIGHT?

Answers

The distribution is skewed to the right.

How to find distribution is skewed?

If the median is to the left of the center of the box and the right whisker is substantially longer than the left whisker in a box plot, then the distribution is skewed to the right.

This means that the majority of the data is clustered on the left side of the box plot and there are some extreme values on the right side that are causing the right whisker to be longer.

The median being to the left of the center of the box indicates that the data is not symmetric and is pulled to the left by the majority of the values.

Learn more about skewed distribution

brainly.com/question/1604092

#SPJ11

In the figure below, m 24 = 106°. Find mZ1, m/2, and m 23. 1 X 3 2 ترا = 0° m 22 = ° m2 1 = m 23 = °​

Answers

Answer:

m<1=74

m<2=106

m<3=74

Step-by-step explanation:

m<1:

=180-106

=74

m<2:

=106

m<3;

if m<1 is 74, m<s will also be 74

The sum of two numbers is 425. Find the two numbers if 20% of the first number is equal to 30% of the second number.​

Answers

Answer:

Let's call the first number x and the second number y.

From the problem, we know that:

x + y = 425 (the sum of the two numbers is 425)20% of x = 30% of y (20% of the first number is equal to 30% of the second number)

To solve for x and y, we can start by expressing one variable in terms of the other, using one of the equations.

Rearranging the first equation to solve for x, we get:

x = 425 - y

Now we can substitute this expression for x into the second equation, and solve for y:

0.2x = 0.3y (substituting x = 425 - y)

0.2(425 - y) = 0.3y (distributing the 0.2)

85 - 0.2y = 0.3y (combining like terms)

85 = 0.5y (adding 0.2y to both sides)

y = 170 (dividing both sides by 0.5)

So the second number is 170. To find the first number, we can substitute this value back into the first equation:

x + y = 425 (substituting y = 170)

x + 170 = 425

x = 255

Therefore, the two numbers are 255 and 170.

how to check 2(a+3)=-12

Answers

Answer:

a = -9

Step-by-step explanation:

2(a+3) = -12

2a + 6 = -12

2a = -18

a = -9

Let's Check

2(-9 + 3) = -12

2(-6) = -12

-12 = -12

So, a = -9 is the correct answer.

Other Questions
The GSSP concept was introduced in the ______, to provide accepted definitions for ______ units. There is some evidence that pharyngeal gill slits occur in certain species of echinoderms that appear early in the fossil record. If confirmed, what do these data suggest? In the context of entrepreneurial orientation, an organization with a tendency of _____ grants individuals and teams the freedom to exercise their creativity, champion promising ideas, and carry them through to completion.Multiple Choiceinnovativenesscompetitive aggressivenessproactivenessindependent actionrisk taking write the relationship between cells tissue and organs in human body Perez Company is a retail company that specializes in selling outdoor camping equipment. The company is considering opening a new store on October 1, year 1. The company president formed a planning committee to prepare a master budget for the first three months of operation. As budget coordinator, you have been assigned the following tasks.RequiredOctober sales are estimated to be $220,000, of which 45 percent will be cash and 55 percent will be credit. The company expects sales to increase at the rate of 20 percent per month. Prepare a sales budget.The company expects to collect 100 percent of the accounts receivable generated by credit sales in the month following the sale. Prepare a schedule of cash receipts.The cost of goods sold is 60 percent of sales. The company desires to maintain a minimum ending inventory equal to 10 percent of the next months cost of goods sold. However, ending inventory of December is expected to be $13,000. Assume that all purchases are made on account. Prepare an inventory purchases budget.The company pays 80 percent of accounts payable in the month of purchase and the remaining 20 percent in the following month. Prepare a cash payments budget for inventory purchases.Budgeted selling and administrative expenses per month follow:Salary expense (fixed) $ 19,000 Sales commissions 5 % of SalesSupplies expense 2 % of SalesUtilities (fixed) $ 2,400 Depreciation on store fixtures (fixed)* $ 5,000 Rent (fixed) $ 5,800 Miscellaneous (fixed) $ 2,200 *The capital expenditures budget indicates that Perez will spend $210,000 on October 1 for store fixtures, which are expected to have a $30,000 salvage value and a three-year (36-month) useful life.Use this information to prepare a selling and administrative expenses budget.Utilities and sales commissions are paid the month after they are incurred; all other expenses are paid in the month in which they are incurred. Prepare a cash payments budget for selling and administrative expenses.Perez borrows funds, in increments of $1,000, and repays them on the last day of the month. Repayments may be made in any amount available. The company also pays its vendors on the last day of the month. It pays interest of 1 percent per month in cash on the last day of the month. To be prudent, the company desires to maintain a $22,000 cash balance. Prepare a cash budget. A portable hair dryer on the conducts 15-5/8 amperes (1875 watts). What is the effective resistance of the heating element and motor? Assume the 12.5 amperes flows through the heating element, what is the effective resistance of the heating element? How much is the effective resistance of the motor. Note the motor can operate independently of the heating element as a cool down setting. Victor spent $61 on some sandpaper for his modelcars. He bought 2 packages of the smallest-grainsandpaper and spent the rest on the largest-grainsandpaper. How many packages of the largest-grain sandpaper did he buy? What is 15426.82 rounded to the nearest hundredth True/False? One of the ways to represent superclass and subclass relationships when mapping EER Model Diagrams into Relational Schema is by creating a single relation for the superclass that also includes all of the subclass attributes, along with a single discriminating attribute that indicates which subclass each tuple is contained. This method works whether the superclass whether the relationship is disjoint or overlapping When corporate tax rates decline, the net cost of debt financing:A) decreasesB) is unchangedC) increasesD) doubles A genetic mutation causes the three-dimensional shape of a photosynthetic enzyme to change, but the active site is unaffected. Which of the following is/are likely the result(s)? Click or tap a choice to answer the question. There would be no effect on normal enzyme function. This enzyme's shape is not affected by pH. The associated metabolic reaction takes The enzyme now must use an induced fit reaction. more energy The substrate molecules cannot attach to the enzyme. Select all the knee ligaments that are deep to the articular capsule and prevent anterior and posterior movement of the femur on the tibia Check All That Apply Fibular collateral ligament Tibial collateral ligament Anterior and posterior cruciate ligaments Lateral and medial menisch While serving as a subject in a laboratory sleep study for several nights, Steve was awakened whenever he entered REM sleep. It is MOST accurate to state that Steve was in a study designed to examine the effects of Describe the three different types of bipedalism. For each one, be sure to discuss the frequency of bipedal locomotion, describe the extent of bipedal adaptations, provide at least one sample primate who practices this type of bipedalism, and describe why the sample primate uses this form of bipedalism Which of the following circumstances most likely would cause the auditor to suspect that there are material misstatements in the entity's financial statements? a) The entity's management places no emphasis on meeting publicized earnings projections. b) Significant differences between the physical inventory count and the accounting records are not investigated. c) Monthly bank reconciliations ordinarily include several large outstanding checks. d) Cash transactions are electronically processed and recorded, leaving no paper audit trail. John will donate 20$ to the charity, if active change to passive if passive change to active Hoang has worked as a nurse at Springfield General Hospital for 5 years longer than her friend Bill. Two years ago, she had been at the hospital for twice as long. How long has each been at the hospital? When considering the relationship between standard free energy change and the equilibrium constant, if K>1, _______. Select the correct answer below: products are more abundant at equilibrium ?G? is positive ?G? is zero reactants are more abundant at equilibrium when carl first attended college, he either barely passed or failed most of his courses despite his efforts to do well. after working at a low paying job for a couple of years, carl has returned to school. overwhelmed with the demands of exams, term papers, library assignments, and the need to concentrate on his studies, carl finds himself procrastinating and engaging in self-defeating passive behaviors. this example illustrates the pehenomenon _____ true/false. strategy decisions involve issues such as the number and location of warehouses and the selection of appropriate transportation modes.